AAMC CBT6 and 6R OFFICIAL Q&A

This forum made possible through the generous support of SDN members, donors, and sponsors. Thank you.

Vihsadas

No summer
Moderator Emeritus
Lifetime Donor
10+ Year Member
5+ Year Member
15+ Year Member
Joined
Oct 17, 2007
Messages
5,474
Reaction score
56
This is the official Q&A thread for AAMC CBT6 and 6R.

Please post ONLY questions pertaining to AAMC CBT6 and 6R.
Out of respect for people who may not have completed the other exams, do not post questions or material from any other AAMC exam.

Please see this thread for the rules of order before you post.

Good luck on your MCAT!

Members don't see this ad.
 
i would think that you assume its a PURE solvent. if it was already saturated, then there would be a precipitate, wouldnt there be? I haven't read that passage, so maybe I can't understand your question fully.
 
im in the middle of taking 6 right now. The PS wasnt too bad at all. Kind of physics heavy, but the one periodic table passage was a gimme lol. Ok lets see what this verbal was all about. I have been averaging 8's in EK and on AAMC's(I have taken 3-5) except i just took 3, 2 days ago and rocked a 5. hopefully i can pull myself out of this.
 
"The student determines the Ksp of lead (II) bromide to be 4.6 x 10^-6 at 25 degrees celsius."

The question is:

Will lead (II) bromide precipitate if the student mixes 0.001 M solution Pb2+ (aq) with 0.00005 M solution of Br- (aq)?

Answer: No,because the Ksp exceeds the ion product. The ion product is [Pb2+][Br]^2=(0.0001)(0.00005)^2=2.5 x 10^-13. The passage states that the Ksp for lead (II) bromide is 4.6 x 10^-6.

However, my question is, am I to assume that the student added those ions in a solution that was already saturated? And that's why we multiply the concentration of the ions to get the "ion product" since they were on the product side?

PbBr2 (s)--> Pb2+ + 2Br-

Is that how it is?

First of all, the question never states that the solution was saturated. The question was asking you wheter PbBr2 would precipitate given the concentration of the ions (both cation and anions) in solution.

Pretty much the question wanted you find out if Q> Ksp, Q= Ksp, or Q< Ksp. (Q = ion product) If Q = Ksp or Q> then Ksp then the solution will precipitate. If Q<Ksp then it will not.

Since Q = 2=2.5 x 10^-13 and Ksp = 4.6 x 10^-6.
we can see that Q<<<Ksp
and hence the answer is D

I to assume that the student added those ions in a solution that was already saturated? No! If you assume that then adding any amount of ions to a saturated solution will form a precipitate and you will be led to choose the wrong answer.
 
Members don't see this ad :)
Alright so they say compound 1 undergoes a E2 mechanism, but this really confuses me, because doesn't E2 and Sn2 favour primary carbons??? Isn't it E1 and Sn1 that favor the TERTIARY carbons (forming stable carbocations)??

So why are the answers E2 and bimolecular?

- before you answer: "look at the data and what it says in the passage," I ALREADY DID. I did got the right answers to 100 and 101. I just didn't understand why they would be, since the drawing of compound 1 depicts a tertiary bromine.

Dazed & confused.

SS3 :luck:
 
Holy **** I got destroyed by VR in CBT6. Also the Comet passage in PS ate me up. It was more like a VR passage then anything. Take the test in a week. I have lost all my confidence. Sucks.
 
Hey guys, does anyone understand the reasoning to the following question from AAMC 6 below?

#52) Electric power for transmission over long distances is "stepped up" to a very high voltage in order:
A) to produce currents of higher density.
B) to produce higher currents in the transmission wires.
C) to make less insulation necessary.
D) to cut down the heat loss in the transmission wires.
Power transmitted to a load at the user's end of an electrical line is the product of the current I times the voltage V at the user's end. The power company has the option of using a low voltage or a high voltage for a given amount of power delivered. Using Ohm's law (I = V/R), the power delivered can also be written as P = V2/R. The power lost in the transmission line is I2R. Since the resistance R is fixed by the material and dimensions of the line, we see that using a high voltage at a lower current maximizes the power to the user and minimizes the heat loss in the transmission line, choice D.
 
Please do not post AAMC practice test questions without mentioning it in the title. Many of our users have not taken the AAMC tests and seeing questions before taking the test would skew their results.

Merging with the official AAMC 6 thread.
 
If all genotypes are equally fit and if there are no genetic modifiers of the sex ratio trait, what will be the ultimate fate of a population in which 50% of the x chromosomes are currently xi and 50% are xs.
A. Extinction
B. Stable population size, with predominance of females
C. Stable population size with all individuals producing a 50 50 sex ratio
D. Stable population size, with some individuals producing an excess of females and some producing an excess of males.

My answer was B. The answer key says A, but isn't extinction a clear impossibility given that ALL GENOTYPES are equally fit? This is just confusing. :thumbdown:thumbdown:thumbdown:

also, did anyone else think this aamc practice test was loaded with gotcha type questions in PS and BS, typically the types of questions you find on the VR section?

If anyone has an explanation for this question, I'd love to hear it. After reviewing my mistakes, this is the only question on the exam I don't understand at all. I also put B, but that was mostly because B is my default "I have no idea" answer. A was the only answer I had ruled out.

Edit: Ok I have an explanation, but I'm not sure if it's right, so if anyone can correct me that'd be great. The passage says if none of the Xi-bearing genotypes (XiY, XiXi, XiXs) are selected against, then the frequency of Xi is expected to increase to 100%, which means eventually only female offspring would be created, and the population would be extinct because the ladies would have no male fruitflies to bang. The issue I have is that it says you're starting with 50% Xi and 50% Xs, which means there are still be XsXs and XsY fruitflies to mate with eachother in the F1 generation and in future generation.... but I guess gradually their numbers would give way to the Xi and would become extinct?
 
Last edited:
Yeah still really dont understand why you dont take into consideration an angle for that problem you are talking about..

The angle is unnecessary information given to trick you. You know the speed of light, and they give the time it takes to travel betweenthe transmitter and the receiver. They only want the distance between the transmitter and the receiver, so it's a simple d=vt calculatoin. You'd only need the angle if they wanted you to calculate how high the satellite was.
 
I thought the PS section was so easy but my score was the same as always!! wtf... those passages were way to short and simple to be representative of the real test! all basic concepts
 
PS Q 26. A ball is thrown up vertically with velocity v from earth, and returns in time t. If g were reduced to g/6, then t would.... answer: increase by 6

I understand the answer but when I was doing the test I used the equation d=at^2 bc they were throwing the ball upward and it has both time and acc.... using that equation u get that it would increase by squareroot of 6.

Why doesnt that equation work??

Also for the kinematics question involving GPS at 40 degree angle, how do you know when to take the angle into account or not??

I am obviously having trouble with kinematics bc i always use the wrong equations!
 
you gotta use 0 = Vit - 1/2 at^2...

since the displacement is not d, but 0... and you throw up will Vi... and it falls with a negative a... therefore, Vi t = 1/2 at^2 , cancel the t's

Vi = .5 a t

g/6 means t times 6... if Vi don't change.
 
Members don't see this ad :)
Question asks about which atom has the largest Electron Affinity. The answer was Fluorine. I chose Mg. I know F really wants that last electron but when it gets it, i thought it gives off heat and thus EA is large but negative. I chose Mg since it does not wanna lose its s shell and thus a large positive amount of energy is needed.

Someone help me out here? Is EA just absolute numbers? I used TPR and they explicity say negative for F

Thanks in advance

sv
 
Heat released means an exothermic and thus a generally favorable reaction.

Right. I thought that is what happens when F gets an electron, thus becoming an octet. So since its a release of energy, i thought the electron affinity value would be a negative. So i didnt choose it since the question asked for the highest EA. I guess i should just think of this in absolute values.........

thanks for the reply

Steve
 
It seems like kind of poor semantics, but think about it logically. I'm not sure what the units of electron affinity actually are, but I don't think it's in kJ/mol heat released when an electron is gained. Higher electron affinity = more affinity for an electron. Fluorine would definitely have more affinity to gain an electron than magnesium would.
 
It seems like kind of poor semantics, but think about it logically. I'm not sure what the units of electron affinity actually are, but I don't think it's in kJ/mol heat released when an electron is gained. Higher electron affinity = more affinity for an electron. Fluorine would definitely have more affinity to gain an electron than magnesium would.

yeah totally agreed. I just kept thinking its exo and therefore negative but at least now i know to think of it as positive
 
For some reason this just is not clicking with me. So basically, HCl -> [H+] [Cl-], and the Pb(OH)2 -> [Pb2+] [OH-]^2 and when the OH reacts with H+ causing a forward shift, meaning more HCl ions to be produced. So that makes HCl more soluble in Pb(OH)2, but does that mean Pb(OH)2 is more HCl, might seem like a dumb question but its a mixed up in my head. lol


Which of the following substances is most likely to be more soluble in 1.0 M HCl than in 1.0 M NaOH?
A ) AgCl
B ) Pb(OH)2
When an ionic substance dissolves, it dissociates into its constituent ions. The solubility of any substance whose anion is basic will be increased in solutions of low pH, as the basic anion reacts with the H+ in solution and is drawn out of the equilibrium. As the basic anion is consumed, the dissolution reaction is driven to the right by Le Chattelier’s principle. Of the substances given as choices, Pb(OH)2 has the most basic anion, OH&#61485;. Thus it is most likely to show increased solubility in an HCl solution. Answer choice B is the best choice.


C ) CaF2
D ) HI
 
Are you asking if Ph(OH)2 is soluble in HCl? A solution is an equilibrium so forwards=backwards. If there was a little Pb(OH)2 in a lot of HCl it would completely dissociate. The more Pb(OH)2 you add the more of it dissolves until you have too much for the HCl to handle. In that case the HCl would be the thing that completely dissociates and you would have excess Pb(OH)2.

Does that make sense? Chemistry's not my strong suite. AAMC 6 kicked my ass.


For some reason this just is not clicking with me. So basically, HCl -> [H+] [Cl-], and the Pb(OH)2 -> [Pb2+] [OH-]^2 and when the OH reacts with H+ causing a forward shift, meaning more HCl ions to be produced. So that makes HCl more soluble in Pb(OH)2, but does that mean Pb(OH)2 is more HCl, might seem like a dumb question but its a mixed up in my head. lol


Which of the following substances is most likely to be more soluble in 1.0 M HCl than in 1.0 M NaOH?
A &nbsp;) AgCl
B &nbsp;) Pb(OH)2
When an ionic substance dissolves, it dissociates into its constituent ions. The solubility of any substance whose anion is basic will be increased in solutions of low pH, as the basic anion reacts with the H+ in solution and is drawn out of the equilibrium. As the basic anion is consumed, the dissolution reaction is driven to the right by Le Chattelier’s principle. Of the substances given as choices, Pb(OH)2 has the most basic anion, OH&#61485;. Thus it is most likely to show increased solubility in an HCl solution. Answer choice B is the best choice.


C &nbsp;) CaF2
D &nbsp;) HI
 
I'm having a hard time understanding these two questions from Passage 7 BS. Can someone please explain? Thanks! :laugh:

130) The passage suggests that the expression of disease genes probably is important in regulating normal celular physiology because:
A) Altered expression of disease genes leads to disease. (Correct)
B) The expression of disease genes leads to disease. (I chose this one.)

First off, the question seems a bit off to me. Why would expression of disease genes regulate cellular physiology? I understand how the altered expression of disease genes may lead to disease, but wouldn't the expression of disease genes also lead to disease?

132) The most likely shared feature of the clinical manifestations of NF1 is that they all:
C) Arise for one genetic defect ( I chose this one)
D) Arise from defects in gene expression. (Correct)

It states in the passage that "defects in a single gene cause the various clinical feasture that characterize this disease..." Therefore wouldn't C be correct?
 
I'm having a hard time understanding these two questions from Passage 7 BS. Can someone please explain? Thanks! :laugh:

130) The passage suggests that the expression of disease genes probably is important in regulating normal celular physiology because:
A) Altered expression of disease genes leads to disease. (Correct)
B) The expression of disease genes leads to disease. (I chose this one.)

First off, the question seems a bit off to me. Why would expression of disease genes regulate cellular physiology? I understand how the altered expression of disease genes may lead to disease, but wouldn't the expression of disease genes also lead to disease?

132) The most likely shared feature of the clinical manifestations of NF1 is that they all:
C) Arise for one genetic defect ( I chose this one)
D) Arise from defects in gene expression. (Correct)

It states in the passage that "defects in a single gene cause the various clinical feasture that characterize this disease..." Therefore wouldn't C be correct?
read carefully
 
Doesn't the glycerine also exhibit intRAmolecular h-bonding as well?
wouldn't this decrease the BP of glycerine?

thanks
 
I was under the impression that blood flows, like all fluids, must flow down a pressure gradient, meaning that, in order for blood to reach the lower parts of the body, the arterial pressure in the upper body (ex/ arm), must be greater than the arterial pressure in the lower body (ex/ leg).

Any help on this one?
 
Well, glycerine can definitely internally hydrogen bond, so I see where you're coming from. However, I think it's reasonable to assume that a fair number of hydrogen bonds will be made intermolecularly. In addition to that, you have a much higher MW for glycerine than you do for isopropanol.
 
Well, glycerine can definitely internally hydrogen bond, so I see where you're coming from. However, I think it's reasonable to assume that a fair number of hydrogen bonds will be made intermolecularly. In addition to that, you have a much higher MW for glycerine than you do for isopropanol.



sounds like a fair bit of speculation; not that I disagree, though.
 
It's definitely speculation. It's also plausible that glycerine would maintain a relatively tight set of intramolecular hydrogen bonds. Not the greatest question, but I think I would agree with glycerine still probably being the best choice. (It is higher BP in actuality at any rate).
 
#117 of BS from CBT:
if steves blood pressure was higher than joes (everything else being equal), whats the diff in their urinary system function

ans:steves glomerular filtration rate would increase more
my ans: steves reabsorption rate per mL of glomerular filtrate by the peritubular capilaries would be higher

help?
 
Hey I have a question about #31 in the PS section...

I thought when dealing with specific heats, the units of mass is GRAMS as opposed to kilograms...

In this question it says the units of the specific heat of a solid is J/(Kg K)...

Thanks!
 
#117 of BS from CBT:
if steves blood pressure was higher than joes (everything else being equal), whats the diff in their urinary system function

ans:steves glomerular filtration rate would increase more
my ans: steves reabsorption rate per mL of glomerular filtrate by the peritubular capilaries would be higher

help?

Renal filtration works to take things OUT of the body. If you don't send something into the renal filtration system, it won't be leaving (simplification). So Steve is sending more substance into the renal system, as his higher blood pressure forces more substance through the glomerulus. Reabsorption of this stuff has a maximum rate that can be exceeded, in which case you're just excreting nutrients from the body.
 
Renal filtration works to take things OUT of the body. If you don't send something into the renal filtration system, it won't be leaving (simplification). So Steve is sending more substance into the renal system, as his higher blood pressure forces more substance through the glomerulus. Reabsorption of this stuff has a maximum rate that can be exceeded, in which case you're just excreting nutrients from the body.


think of something being 'filtered' as something that becomes 'filtrate'. filtrate is, ultimately, what gets excreted.
 
read carefully

I too am having a hard time understanding the logic behind this question.. Can anyone chime in? I understand why B is wrong, which I also chose originally. I think its wrong because though it may be true, it doesn't answer the question. But I'm trying to sort out how A answers it.

Is the question trying to ask, "Normally, altered genes leads to disease if expressed, but what's significantly different about NF1 to make it abnormal? So the answer A is saying, this is abnormal because it's the altered gene of a disease gene leads to multiple diseases which is why they are so confused in the passage?

Originally Posted by puffylover
I'm having a hard time understanding these two questions from Passage 7 BS. Can someone please explain? Thanks! :laugh:

130) The passage suggests that the expression of disease genes probably is important in regulating normal celular physiology because:
A) Altered expression of disease genes leads to disease. (Correct)
B) The expression of disease genes leads to disease. (I chose this one.)
 
Don't get #113 in BS. I got it right, but because I had to guess and C just seemed like the best answer, but not fully correct. Here it is:

Double-stranded DNA can adfopt one of three helical conformations depending on the nucleotide makeup of the molecule and the amount of hydration. The nucleotide base pairs in a DNA helix are arranged like steps in a spiral staircase. Each one is rotated a few degrees from the previous base pair.

Table 1 AVerage Helical Twist between Adjacent Nucleotide Pairs (Mean and standard Deviation, in Degrees)
Conformation Helical twist
A 33.1 +/- 6
B 35.9 +/- 4
Z 29.9 +/- 1

Investigating the properties of a strand of DNA, researchers determined that there were 12 nucleotide base pairs for every complete 360 degree turn of the helix. The conformation of the DNA strand was:

A) A, not B or Z
B) B, not A or Z
C) Z, not A or B
D) A or B, not Z


I picked C just because Z absolute was a possibility for being the conformation of the helix but A also was within range. Am I adding wrong or something? I hate questions like this... it's these that I have to spend extra time on wondering whether its just a dumb question or whether I'm missing something that make me run short on time towards the end of the question.
 
Don't understand scoring..

So on the paper section, I did the whole test for physical and verbal. For verbal there are 60 questions(the paper version is longer) and I missed 7-which brought my score to a 12. The thing is, the highest the Verbal is scored is a 12! It states on the back that a 12 is basically 51 and up. Is this a good indicator of how I may do on the CBT? Would you say that missing 7 and getting a 12 on the paper version would probably translate to a 14 on the CBT (because the 12 is highest on here, has more questions etc)- I ask this because I suspect that the paper test that was created wasnt an accurate predictor of how one would do on the real thing, I could be wrong

Also, on physical sciences I got a 12 on the paper version. Can i expect a higher on the CBT? Again the highest on the paper version was a 12.
 
I thoroughly enjoy the number of insane math errors I make. For #113 I divided 360 by 12 and got 33. So I initially picked D. However, since it was a math problem, I marked it and came back at the end to check my math. I switched to C when I noticed my mistake.

I suppose the key is standard deviation reflects the range of experimental results. The actual geometry presumably is the mean of all the experimental results. So conformation A is actually 33, though the investigators only arrive at that conclusion after many experiments. Then I think you are supposed to assume the mystery strand was determined to actually be 30 after many experiments as well.
 
Don't get #113 in BS. I got it right, but because I had to guess and C just seemed like the best answer, but not fully correct. Here it is:

Double-stranded DNA can adfopt one of three helical conformations depending on the nucleotide makeup of the molecule and the amount of hydration. The nucleotide base pairs in a DNA helix are arranged like steps in a spiral staircase. Each one is rotated a few degrees from the previous base pair.

Table 1 AVerage Helical Twist between Adjacent Nucleotide Pairs (Mean and standard Deviation, in Degrees)
Conformation Helical twist
A 33.1 +/- 6
B 35.9 +/- 4
Z 29.9 +/- 1

Investigating the properties of a strand of DNA, researchers determined that there were 12 nucleotide base pairs for every complete 360 degree turn of the helix. The conformation of the DNA strand was:

A) A, not B or Z
B) B, not A or Z
C) Z, not A or B
D) A or B, not Z


I picked C just because Z absolute was a possibility for being the conformation of the helix but A also was within range. Am I adding wrong or something? I hate questions like this... it's these that I have to spend extra time on wondering whether its just a dumb question or whether I'm missing something that make me run short on time towards the end of the question.

You are correct. You KNOW Z is the best answer, but there are no choices for "Z or A, but not B" so it MUST be C since it is the only one that does not exclude Z.
 
A person suffering an asthmatic attack often has more difficulty exhaling than inhaling; the action of leukotrienes on bronchial smooth muscle contributes to this difficulty. As a result, what primary effect do leukotrienes have on lung gas exchange?

A) No effect, because bronchial smooth muscle is not found on alveoli
B) An increased accumulation of O2, leading to respiratory acidosis
C) An increased accumulation of CO2, leading to respiratory acidosis
D) A decreased intake of O2, leading to metabolic acidosis

Answer: C


OK, I understand there would be an accumulation of CO2 leading to respiratory acidosis, but the question italicizes PRIMARY. How is the the constriction of bronchial smooth muscle leading to an accumulation of CO2 a primary effect on lung gas exchange? Isn't that a secondary effect? I'd think a primary effect on lung gas exchange would be something directly affecting the diffusion of gasses between the alveoli and capillaries.
 
A person suffering an asthmatic attack often has more difficulty exhaling than inhaling; the action of leukotrienes on bronchial smooth muscle contributes to this difficulty. As a result, what primary effect do leukotrienes have on lung gas exchange?

A) No effect, because bronchial smooth muscle is not found on alveoli
B) An increased accumulation of O2, leading to respiratory acidosis
C) An increased accumulation of CO2, leading to respiratory acidosis
D) A decreased intake of O2, leading to metabolic acidosis

Answer: C


OK, I understand there would be an accumulation of CO2 leading to respiratory acidosis, but the question italicizes PRIMARY. How is the the constriction of bronchial smooth muscle leading to an accumulation of CO2 a primary effect on lung gas exchange? Isn't that a secondary effect? I'd think a primary effect on lung gas exchange would be something directly affecting the diffusion of gasses between the alveoli and capillaries.

That's exactly what C says.
 
P.S.: Did anyone else get completely destroyed by the last VR passage? I only had 6 minutes to read it and knock out the questions and found myself totally lost while reading.

Who actually studies ice tectonics anyway?? :sleep:
 
What was like the general feeling for the Verbal section for this CBT? I took it yesterday and got a 10. I thought it was pretty straightforward... Any thoughts?
 
I thought it was pretty straight except for the last passage and the feankenstein one. Nevertheless, a relatively easy VR.
 
Question 164 on BS confused me. They're including the angle between the last nucleotide base pair and first nucleotide base pair a part of the 360 degrees? Like on the surface, this is an easy problem. But when you think about what's actually right, the question is worded badly.
 
Just took this and got a 13-11-11; 35. I wish I could say I was happy with that score, but I'm PISSED about that BS score. Who else felt that the BS was just weird on this? There were a lot of questions that asked you to draw upon pretty trivial knowledge that I would think most people wouldn't know. Also, a pretty lengthy passage with just one question?

Since this thread is 2 years old now, would some of you guys who have since taken the MCAT like to weigh in on the comparison of AAMC6 to the real thing?
 
Question 164 on BS confused me. They're including the angle between the last nucleotide base pair and first nucleotide base pair a part of the 360 degrees? Like on the surface, this is an easy problem. But when you think about what's actually right, the question is worded badly.

I did notice that it was a bit odd problem. Wonder if the real AAMC is like this in terms of ambiguity.

I just looked at the answer choices and saw that the answer should be close to 30 and picked that answer. I am generally hesitant to pick answers that may be right because AAMC always seems to be more clear on what answers are wrong than what answers may be correct (if that makes sense).
 
Since this thread is 2 years old now, would some of you guys who have since taken the MCAT like to weigh in on the comparison of AAMC6 to the real thing?

The PTs on e-mcat are very much like the real thing--including, unfortunately, the unpredictable difficulty.

This probably is not useful information, but my divergence from PT 6 on the real thing in 2008 was PS -2, VS -1, BS +1 (that is, I scored worse on the real PS and VS and better on the real BS)
 
The PTs on e-mcat are very much like the real thing--including, unfortunately, the unpredictable difficulty.

This probably is not useful information, but my divergence from PT 6 on the real thing in 2008 was PS -2, VS -1, BS +1 (that is, I scored worse on the real PS and VS and better on the real BS)

Thanks for the response. You must have had one hell of a PS, because it seems like most people's problems with AAMC6 was the PS. I'm kind of hoping there's going to be a more generous curve on the real thing. I got 34/40 on verbal on this one and it was an 11? That seems a little steep.
 
Top